Postać liczby

Podzielność. Reszty z dzielenia. Kongruencje. Systemy pozycyjne. Równania diofantyczne. Liczby pierwsze i względnie pierwsze. NWW i NWD.
szymek12
Użytkownik
Użytkownik
Posty: 659
Rejestracja: 24 kwie 2008, o 20:15
Płeć: Mężczyzna
Lokalizacja: Strzyżów
Podziękował: 136 razy
Pomógł: 54 razy

Postać liczby

Post autor: szymek12 »

Wykazać, że każdą liczbę naturalną \(\displaystyle{ n>7}\) można przedstawić w postaci: \(\displaystyle{ n=3a+5b}\), gdzie \(\displaystyle{ a,b \mathbb{N}}\)
Awatar użytkownika
Vax
Użytkownik
Użytkownik
Posty: 2913
Rejestracja: 27 kwie 2010, o 22:07
Płeć: Mężczyzna
Lokalizacja: Biała Podlaska / Warszawa
Podziękował: 4 razy
Pomógł: 612 razy

Postać liczby

Post autor: Vax »

Zauważ, że równanie \(\displaystyle{ 3a+5b=n}\) spełnia dla każdego całkowitego k:

\(\displaystyle{ \begin{cases} a = 2n-5k\\ b = 3k-n \end{cases}}\)

Pozostaje pokazać, że dla \(\displaystyle{ n\ge 8}\) istnieje k, że \(\displaystyle{ a\ge 0 \wedge b\ge 0}\) zauważmy, że:

\(\displaystyle{ 0 \le b = 3k-n \le 3k-8 \Leftrightarrow (k \ jest \ calkowite) \Leftrightarrow k \ge 3}\)

Wobec tego:

\(\displaystyle{ 0 \le a = 2n-5k \le 2n-15 \Leftrightarrow n\ge 8}\) co jest zgodne z założeniem.

Pozdrawiam.
ElEski
Użytkownik
Użytkownik
Posty: 304
Rejestracja: 22 maja 2010, o 17:20
Płeć: Mężczyzna
Lokalizacja: Warszawa
Pomógł: 12 razy

Postać liczby

Post autor: ElEski »

szymek12,
Jak rozumiem, za liczbę naturalną uznajesz też liczbę 0.
No to:
liczby postaci \(\displaystyle{ 3k}\) załatwiasz tak: \(\displaystyle{ a=k}\), \(\displaystyle{ b=0}\)
liczby postaci \(\displaystyle{ 3k+1}\) załatwiasz tak: jakbyś przyjął, że \(\displaystyle{ a=k}\), \(\displaystyle{ b=0}\), to dostajesz wyrażenie o \(\displaystyle{ 1}\) za małe. Zatem musisz ją jakoś powiększyć o \(\displaystyle{ 1}\).
W tym celu \(\displaystyle{ a}\) zmniejszasz o \(\displaystyle{ 3}\), a też \(\displaystyle{ b}\) zwiększasz o \(\displaystyle{ 2}\)
czyli w tym wypadku \(\displaystyle{ a=k-3}\), \(\displaystyle{ b=2}\) (konieczne jest zauważenie, że \(\displaystyle{ k}\) musi być większe od 3.) Można to też zrobić tak, że najmniejsza liczba tej postaci spełniająca warunki zadania ma właśnie \(\displaystyle{ a=k-3}\), \(\displaystyle{ b=2}\), a każda kolejna powstaje przez powiększenie \(\displaystyle{ a}\) o \(\displaystyle{ 1}\).

Analogicznie, kiedy liczba jest postaci\(\displaystyle{ 3k+2}\)
To trochę inne rozwiązanie, ale to zadanie jest bardzo proste i można milion sposobów wymyślić.
frej

Postać liczby

Post autor: frej »

Znajdź przedstawienie pierwszych trzech liczb a potem zapisz \(\displaystyle{ n=3+(n-3)}\) i założenie indukcyjne.
ODPOWIEDZ